Prove that the rank of $(1-I)$ is $n$












1












$begingroup$


The rank of $(1-I_n)$, where $1$ is the $n times n$ all-1 matrix and $I_n$ the $n times n$ identity matrix, seems to be $n$.



How to prove this concisely?










share|cite|improve this question











$endgroup$








  • 2




    $begingroup$
    Well if you look at a example. When you subtract a matrix filled with ones from a matrix with 1`s going down in a diagonal this creates all columns that are linearly independent due to the zeroes appearing one position lower from column 1 to column N.
    $endgroup$
    – user60887
    Jun 20 '13 at 18:11










  • $begingroup$
    Maybe easiest is to assume given that $r(A + B) leq r(A) + r(B)$ (subadditivity of rank)? By then choosing $A = 1$ and $B = -(1-I)$, this gives $r(I) leq r(1) + r(-(1-I))$ or $n leq 1 + r(-(1-I))$ and hence $r(1-I) = r(-(1-I)) geq n-1$.
    $endgroup$
    – ranky
    Jun 20 '13 at 18:24
















1












$begingroup$


The rank of $(1-I_n)$, where $1$ is the $n times n$ all-1 matrix and $I_n$ the $n times n$ identity matrix, seems to be $n$.



How to prove this concisely?










share|cite|improve this question











$endgroup$








  • 2




    $begingroup$
    Well if you look at a example. When you subtract a matrix filled with ones from a matrix with 1`s going down in a diagonal this creates all columns that are linearly independent due to the zeroes appearing one position lower from column 1 to column N.
    $endgroup$
    – user60887
    Jun 20 '13 at 18:11










  • $begingroup$
    Maybe easiest is to assume given that $r(A + B) leq r(A) + r(B)$ (subadditivity of rank)? By then choosing $A = 1$ and $B = -(1-I)$, this gives $r(I) leq r(1) + r(-(1-I))$ or $n leq 1 + r(-(1-I))$ and hence $r(1-I) = r(-(1-I)) geq n-1$.
    $endgroup$
    – ranky
    Jun 20 '13 at 18:24














1












1








1


1



$begingroup$


The rank of $(1-I_n)$, where $1$ is the $n times n$ all-1 matrix and $I_n$ the $n times n$ identity matrix, seems to be $n$.



How to prove this concisely?










share|cite|improve this question











$endgroup$




The rank of $(1-I_n)$, where $1$ is the $n times n$ all-1 matrix and $I_n$ the $n times n$ identity matrix, seems to be $n$.



How to prove this concisely?







linear-algebra matrices






share|cite|improve this question















share|cite|improve this question













share|cite|improve this question




share|cite|improve this question








edited Jun 20 '13 at 18:57









Davide Giraudo

128k17156268




128k17156268










asked Jun 20 '13 at 18:02









rankyranky

61




61








  • 2




    $begingroup$
    Well if you look at a example. When you subtract a matrix filled with ones from a matrix with 1`s going down in a diagonal this creates all columns that are linearly independent due to the zeroes appearing one position lower from column 1 to column N.
    $endgroup$
    – user60887
    Jun 20 '13 at 18:11










  • $begingroup$
    Maybe easiest is to assume given that $r(A + B) leq r(A) + r(B)$ (subadditivity of rank)? By then choosing $A = 1$ and $B = -(1-I)$, this gives $r(I) leq r(1) + r(-(1-I))$ or $n leq 1 + r(-(1-I))$ and hence $r(1-I) = r(-(1-I)) geq n-1$.
    $endgroup$
    – ranky
    Jun 20 '13 at 18:24














  • 2




    $begingroup$
    Well if you look at a example. When you subtract a matrix filled with ones from a matrix with 1`s going down in a diagonal this creates all columns that are linearly independent due to the zeroes appearing one position lower from column 1 to column N.
    $endgroup$
    – user60887
    Jun 20 '13 at 18:11










  • $begingroup$
    Maybe easiest is to assume given that $r(A + B) leq r(A) + r(B)$ (subadditivity of rank)? By then choosing $A = 1$ and $B = -(1-I)$, this gives $r(I) leq r(1) + r(-(1-I))$ or $n leq 1 + r(-(1-I))$ and hence $r(1-I) = r(-(1-I)) geq n-1$.
    $endgroup$
    – ranky
    Jun 20 '13 at 18:24








2




2




$begingroup$
Well if you look at a example. When you subtract a matrix filled with ones from a matrix with 1`s going down in a diagonal this creates all columns that are linearly independent due to the zeroes appearing one position lower from column 1 to column N.
$endgroup$
– user60887
Jun 20 '13 at 18:11




$begingroup$
Well if you look at a example. When you subtract a matrix filled with ones from a matrix with 1`s going down in a diagonal this creates all columns that are linearly independent due to the zeroes appearing one position lower from column 1 to column N.
$endgroup$
– user60887
Jun 20 '13 at 18:11












$begingroup$
Maybe easiest is to assume given that $r(A + B) leq r(A) + r(B)$ (subadditivity of rank)? By then choosing $A = 1$ and $B = -(1-I)$, this gives $r(I) leq r(1) + r(-(1-I))$ or $n leq 1 + r(-(1-I))$ and hence $r(1-I) = r(-(1-I)) geq n-1$.
$endgroup$
– ranky
Jun 20 '13 at 18:24




$begingroup$
Maybe easiest is to assume given that $r(A + B) leq r(A) + r(B)$ (subadditivity of rank)? By then choosing $A = 1$ and $B = -(1-I)$, this gives $r(I) leq r(1) + r(-(1-I))$ or $n leq 1 + r(-(1-I))$ and hence $r(1-I) = r(-(1-I)) geq n-1$.
$endgroup$
– ranky
Jun 20 '13 at 18:24










4 Answers
4






active

oldest

votes


















2












$begingroup$

Let's call $J$ the matrix with all coefficients equal to $1$. Its eigenvalues are $n$ and $0$: in fact $J$ has rank $1$ and so $0$ is an eigenvalue of multiplicity $n-1$; clearly $Jv=nv$ where $v$ is the "all $1$" vector.



So $1$ is not a root of the characteristic polynomial of $J$, that is,
$$
det(J-XI_n)
$$
which means that $det(J-I_n)ne0$.



Of course we assume $n>1$, otherwise the assertion is false.






share|cite|improve this answer









$endgroup$





















    2












    $begingroup$

    If the rank were not to be $n$, there exists a non-zero vector $x$ such that
    $$(ee^T - I)x = 0$$
    i.e.,
    $$sum_{overset{i=1}{i neq j}}^n x_i = 0$$ for all $j in {1,2,ldots,n}$. Can this be true for $x neq 0$ ?



    Also, as an aside, the inverse of $(ee^T - I)$ is given by the Sherman-Morrison formula or the more general Woodbury formula
    $$- left(I + dfrac{ee^T}{1-e^Te}right) = - left(I - dfrac{ee^T}{n-1}right)$$



    In general, if $A$ is invertible, then $A+uv^T$ is invertible when $1+v^TA^{-1}u neq 0$. In your case, this corresponds to the condition that $n-1 neq 0$, i.e., $n neq 1$.






    share|cite|improve this answer









    $endgroup$





















      0












      $begingroup$

      If $A$ is any $ntimes n$ matrix of rank$~1$, then it has an eigenspace for$~0$ of dimension$~n-1$, and the eigenvalue$~0$ has at least that (algebraic) multiplicity; the final eigenvalue is $deftr{operatorname{tr}}tr A$ (as that is the sum of the eigenvalues). Therefore $A-lambda I$ with $lambdaneq0$ is invertible unless $lambda=tr A$, in which case $defrk{operatorname{rk}}rk(A-lambda I)=n-1$. In your case if $A$ is the matrix you confusingly call$~1$, you have $tr A=n$, so for $lambda=1$ you get that $A-I$ is invertible unless $n=1$, in which case it has rank$~n-1=0$.






      share|cite|improve this answer











      $endgroup$





















        0












        $begingroup$

        $newcommand{rank}{operatorname{rank}}$Hint: The matrix $A=[a_{ij}]_{ntimes n}$is full column rank ($rank(A)=n$)if and only if it's invertible.



        $$det(1-I_n)=(n-1)(-1)^{n-1}neq0$$



        use row echelon operations to Compute $det(B)$$$forall i, ~~ 2le ile n :~~~~~~~C _1-C _i,~~~~~~ R_1+R_i$$



        wherein $R$ stands for row and $C$ stands for column. Now;$$det(B)=detbegin{pmatrix}
        r & a & a & cdots &a \
        a & r& a & cdots & a \
        vdots & vdots& vdots & ddots & vdots \
        a & a & a & cdots & r
        end{pmatrix}=(r+(n-1)a)(r-a)^{n-1}$$






        share|cite|improve this answer











        $endgroup$














          Your Answer





          StackExchange.ifUsing("editor", function () {
          return StackExchange.using("mathjaxEditing", function () {
          StackExchange.MarkdownEditor.creationCallbacks.add(function (editor, postfix) {
          StackExchange.mathjaxEditing.prepareWmdForMathJax(editor, postfix, [["$", "$"], ["\\(","\\)"]]);
          });
          });
          }, "mathjax-editing");

          StackExchange.ready(function() {
          var channelOptions = {
          tags: "".split(" "),
          id: "69"
          };
          initTagRenderer("".split(" "), "".split(" "), channelOptions);

          StackExchange.using("externalEditor", function() {
          // Have to fire editor after snippets, if snippets enabled
          if (StackExchange.settings.snippets.snippetsEnabled) {
          StackExchange.using("snippets", function() {
          createEditor();
          });
          }
          else {
          createEditor();
          }
          });

          function createEditor() {
          StackExchange.prepareEditor({
          heartbeatType: 'answer',
          autoActivateHeartbeat: false,
          convertImagesToLinks: true,
          noModals: true,
          showLowRepImageUploadWarning: true,
          reputationToPostImages: 10,
          bindNavPrevention: true,
          postfix: "",
          imageUploader: {
          brandingHtml: "Powered by u003ca class="icon-imgur-white" href="https://imgur.com/"u003eu003c/au003e",
          contentPolicyHtml: "User contributions licensed under u003ca href="https://creativecommons.org/licenses/by-sa/3.0/"u003ecc by-sa 3.0 with attribution requiredu003c/au003e u003ca href="https://stackoverflow.com/legal/content-policy"u003e(content policy)u003c/au003e",
          allowUrls: true
          },
          noCode: true, onDemand: true,
          discardSelector: ".discard-answer"
          ,immediatelyShowMarkdownHelp:true
          });


          }
          });














          draft saved

          draft discarded


















          StackExchange.ready(
          function () {
          StackExchange.openid.initPostLogin('.new-post-login', 'https%3a%2f%2fmath.stackexchange.com%2fquestions%2f425597%2fprove-that-the-rank-of-1-i-is-n%23new-answer', 'question_page');
          }
          );

          Post as a guest















          Required, but never shown

























          4 Answers
          4






          active

          oldest

          votes








          4 Answers
          4






          active

          oldest

          votes









          active

          oldest

          votes






          active

          oldest

          votes









          2












          $begingroup$

          Let's call $J$ the matrix with all coefficients equal to $1$. Its eigenvalues are $n$ and $0$: in fact $J$ has rank $1$ and so $0$ is an eigenvalue of multiplicity $n-1$; clearly $Jv=nv$ where $v$ is the "all $1$" vector.



          So $1$ is not a root of the characteristic polynomial of $J$, that is,
          $$
          det(J-XI_n)
          $$
          which means that $det(J-I_n)ne0$.



          Of course we assume $n>1$, otherwise the assertion is false.






          share|cite|improve this answer









          $endgroup$


















            2












            $begingroup$

            Let's call $J$ the matrix with all coefficients equal to $1$. Its eigenvalues are $n$ and $0$: in fact $J$ has rank $1$ and so $0$ is an eigenvalue of multiplicity $n-1$; clearly $Jv=nv$ where $v$ is the "all $1$" vector.



            So $1$ is not a root of the characteristic polynomial of $J$, that is,
            $$
            det(J-XI_n)
            $$
            which means that $det(J-I_n)ne0$.



            Of course we assume $n>1$, otherwise the assertion is false.






            share|cite|improve this answer









            $endgroup$
















              2












              2








              2





              $begingroup$

              Let's call $J$ the matrix with all coefficients equal to $1$. Its eigenvalues are $n$ and $0$: in fact $J$ has rank $1$ and so $0$ is an eigenvalue of multiplicity $n-1$; clearly $Jv=nv$ where $v$ is the "all $1$" vector.



              So $1$ is not a root of the characteristic polynomial of $J$, that is,
              $$
              det(J-XI_n)
              $$
              which means that $det(J-I_n)ne0$.



              Of course we assume $n>1$, otherwise the assertion is false.






              share|cite|improve this answer









              $endgroup$



              Let's call $J$ the matrix with all coefficients equal to $1$. Its eigenvalues are $n$ and $0$: in fact $J$ has rank $1$ and so $0$ is an eigenvalue of multiplicity $n-1$; clearly $Jv=nv$ where $v$ is the "all $1$" vector.



              So $1$ is not a root of the characteristic polynomial of $J$, that is,
              $$
              det(J-XI_n)
              $$
              which means that $det(J-I_n)ne0$.



              Of course we assume $n>1$, otherwise the assertion is false.







              share|cite|improve this answer












              share|cite|improve this answer



              share|cite|improve this answer










              answered Jun 20 '13 at 18:07









              egregegreg

              185k1486207




              185k1486207























                  2












                  $begingroup$

                  If the rank were not to be $n$, there exists a non-zero vector $x$ such that
                  $$(ee^T - I)x = 0$$
                  i.e.,
                  $$sum_{overset{i=1}{i neq j}}^n x_i = 0$$ for all $j in {1,2,ldots,n}$. Can this be true for $x neq 0$ ?



                  Also, as an aside, the inverse of $(ee^T - I)$ is given by the Sherman-Morrison formula or the more general Woodbury formula
                  $$- left(I + dfrac{ee^T}{1-e^Te}right) = - left(I - dfrac{ee^T}{n-1}right)$$



                  In general, if $A$ is invertible, then $A+uv^T$ is invertible when $1+v^TA^{-1}u neq 0$. In your case, this corresponds to the condition that $n-1 neq 0$, i.e., $n neq 1$.






                  share|cite|improve this answer









                  $endgroup$


















                    2












                    $begingroup$

                    If the rank were not to be $n$, there exists a non-zero vector $x$ such that
                    $$(ee^T - I)x = 0$$
                    i.e.,
                    $$sum_{overset{i=1}{i neq j}}^n x_i = 0$$ for all $j in {1,2,ldots,n}$. Can this be true for $x neq 0$ ?



                    Also, as an aside, the inverse of $(ee^T - I)$ is given by the Sherman-Morrison formula or the more general Woodbury formula
                    $$- left(I + dfrac{ee^T}{1-e^Te}right) = - left(I - dfrac{ee^T}{n-1}right)$$



                    In general, if $A$ is invertible, then $A+uv^T$ is invertible when $1+v^TA^{-1}u neq 0$. In your case, this corresponds to the condition that $n-1 neq 0$, i.e., $n neq 1$.






                    share|cite|improve this answer









                    $endgroup$
















                      2












                      2








                      2





                      $begingroup$

                      If the rank were not to be $n$, there exists a non-zero vector $x$ such that
                      $$(ee^T - I)x = 0$$
                      i.e.,
                      $$sum_{overset{i=1}{i neq j}}^n x_i = 0$$ for all $j in {1,2,ldots,n}$. Can this be true for $x neq 0$ ?



                      Also, as an aside, the inverse of $(ee^T - I)$ is given by the Sherman-Morrison formula or the more general Woodbury formula
                      $$- left(I + dfrac{ee^T}{1-e^Te}right) = - left(I - dfrac{ee^T}{n-1}right)$$



                      In general, if $A$ is invertible, then $A+uv^T$ is invertible when $1+v^TA^{-1}u neq 0$. In your case, this corresponds to the condition that $n-1 neq 0$, i.e., $n neq 1$.






                      share|cite|improve this answer









                      $endgroup$



                      If the rank were not to be $n$, there exists a non-zero vector $x$ such that
                      $$(ee^T - I)x = 0$$
                      i.e.,
                      $$sum_{overset{i=1}{i neq j}}^n x_i = 0$$ for all $j in {1,2,ldots,n}$. Can this be true for $x neq 0$ ?



                      Also, as an aside, the inverse of $(ee^T - I)$ is given by the Sherman-Morrison formula or the more general Woodbury formula
                      $$- left(I + dfrac{ee^T}{1-e^Te}right) = - left(I - dfrac{ee^T}{n-1}right)$$



                      In general, if $A$ is invertible, then $A+uv^T$ is invertible when $1+v^TA^{-1}u neq 0$. In your case, this corresponds to the condition that $n-1 neq 0$, i.e., $n neq 1$.







                      share|cite|improve this answer












                      share|cite|improve this answer



                      share|cite|improve this answer










                      answered Jun 20 '13 at 18:08







                      user17762






























                          0












                          $begingroup$

                          If $A$ is any $ntimes n$ matrix of rank$~1$, then it has an eigenspace for$~0$ of dimension$~n-1$, and the eigenvalue$~0$ has at least that (algebraic) multiplicity; the final eigenvalue is $deftr{operatorname{tr}}tr A$ (as that is the sum of the eigenvalues). Therefore $A-lambda I$ with $lambdaneq0$ is invertible unless $lambda=tr A$, in which case $defrk{operatorname{rk}}rk(A-lambda I)=n-1$. In your case if $A$ is the matrix you confusingly call$~1$, you have $tr A=n$, so for $lambda=1$ you get that $A-I$ is invertible unless $n=1$, in which case it has rank$~n-1=0$.






                          share|cite|improve this answer











                          $endgroup$


















                            0












                            $begingroup$

                            If $A$ is any $ntimes n$ matrix of rank$~1$, then it has an eigenspace for$~0$ of dimension$~n-1$, and the eigenvalue$~0$ has at least that (algebraic) multiplicity; the final eigenvalue is $deftr{operatorname{tr}}tr A$ (as that is the sum of the eigenvalues). Therefore $A-lambda I$ with $lambdaneq0$ is invertible unless $lambda=tr A$, in which case $defrk{operatorname{rk}}rk(A-lambda I)=n-1$. In your case if $A$ is the matrix you confusingly call$~1$, you have $tr A=n$, so for $lambda=1$ you get that $A-I$ is invertible unless $n=1$, in which case it has rank$~n-1=0$.






                            share|cite|improve this answer











                            $endgroup$
















                              0












                              0








                              0





                              $begingroup$

                              If $A$ is any $ntimes n$ matrix of rank$~1$, then it has an eigenspace for$~0$ of dimension$~n-1$, and the eigenvalue$~0$ has at least that (algebraic) multiplicity; the final eigenvalue is $deftr{operatorname{tr}}tr A$ (as that is the sum of the eigenvalues). Therefore $A-lambda I$ with $lambdaneq0$ is invertible unless $lambda=tr A$, in which case $defrk{operatorname{rk}}rk(A-lambda I)=n-1$. In your case if $A$ is the matrix you confusingly call$~1$, you have $tr A=n$, so for $lambda=1$ you get that $A-I$ is invertible unless $n=1$, in which case it has rank$~n-1=0$.






                              share|cite|improve this answer











                              $endgroup$



                              If $A$ is any $ntimes n$ matrix of rank$~1$, then it has an eigenspace for$~0$ of dimension$~n-1$, and the eigenvalue$~0$ has at least that (algebraic) multiplicity; the final eigenvalue is $deftr{operatorname{tr}}tr A$ (as that is the sum of the eigenvalues). Therefore $A-lambda I$ with $lambdaneq0$ is invertible unless $lambda=tr A$, in which case $defrk{operatorname{rk}}rk(A-lambda I)=n-1$. In your case if $A$ is the matrix you confusingly call$~1$, you have $tr A=n$, so for $lambda=1$ you get that $A-I$ is invertible unless $n=1$, in which case it has rank$~n-1=0$.







                              share|cite|improve this answer














                              share|cite|improve this answer



                              share|cite|improve this answer








                              edited Jun 20 '13 at 19:34

























                              answered Jun 20 '13 at 19:26









                              Marc van LeeuwenMarc van Leeuwen

                              88.7k5111230




                              88.7k5111230























                                  0












                                  $begingroup$

                                  $newcommand{rank}{operatorname{rank}}$Hint: The matrix $A=[a_{ij}]_{ntimes n}$is full column rank ($rank(A)=n$)if and only if it's invertible.



                                  $$det(1-I_n)=(n-1)(-1)^{n-1}neq0$$



                                  use row echelon operations to Compute $det(B)$$$forall i, ~~ 2le ile n :~~~~~~~C _1-C _i,~~~~~~ R_1+R_i$$



                                  wherein $R$ stands for row and $C$ stands for column. Now;$$det(B)=detbegin{pmatrix}
                                  r & a & a & cdots &a \
                                  a & r& a & cdots & a \
                                  vdots & vdots& vdots & ddots & vdots \
                                  a & a & a & cdots & r
                                  end{pmatrix}=(r+(n-1)a)(r-a)^{n-1}$$






                                  share|cite|improve this answer











                                  $endgroup$


















                                    0












                                    $begingroup$

                                    $newcommand{rank}{operatorname{rank}}$Hint: The matrix $A=[a_{ij}]_{ntimes n}$is full column rank ($rank(A)=n$)if and only if it's invertible.



                                    $$det(1-I_n)=(n-1)(-1)^{n-1}neq0$$



                                    use row echelon operations to Compute $det(B)$$$forall i, ~~ 2le ile n :~~~~~~~C _1-C _i,~~~~~~ R_1+R_i$$



                                    wherein $R$ stands for row and $C$ stands for column. Now;$$det(B)=detbegin{pmatrix}
                                    r & a & a & cdots &a \
                                    a & r& a & cdots & a \
                                    vdots & vdots& vdots & ddots & vdots \
                                    a & a & a & cdots & r
                                    end{pmatrix}=(r+(n-1)a)(r-a)^{n-1}$$






                                    share|cite|improve this answer











                                    $endgroup$
















                                      0












                                      0








                                      0





                                      $begingroup$

                                      $newcommand{rank}{operatorname{rank}}$Hint: The matrix $A=[a_{ij}]_{ntimes n}$is full column rank ($rank(A)=n$)if and only if it's invertible.



                                      $$det(1-I_n)=(n-1)(-1)^{n-1}neq0$$



                                      use row echelon operations to Compute $det(B)$$$forall i, ~~ 2le ile n :~~~~~~~C _1-C _i,~~~~~~ R_1+R_i$$



                                      wherein $R$ stands for row and $C$ stands for column. Now;$$det(B)=detbegin{pmatrix}
                                      r & a & a & cdots &a \
                                      a & r& a & cdots & a \
                                      vdots & vdots& vdots & ddots & vdots \
                                      a & a & a & cdots & r
                                      end{pmatrix}=(r+(n-1)a)(r-a)^{n-1}$$






                                      share|cite|improve this answer











                                      $endgroup$



                                      $newcommand{rank}{operatorname{rank}}$Hint: The matrix $A=[a_{ij}]_{ntimes n}$is full column rank ($rank(A)=n$)if and only if it's invertible.



                                      $$det(1-I_n)=(n-1)(-1)^{n-1}neq0$$



                                      use row echelon operations to Compute $det(B)$$$forall i, ~~ 2le ile n :~~~~~~~C _1-C _i,~~~~~~ R_1+R_i$$



                                      wherein $R$ stands for row and $C$ stands for column. Now;$$det(B)=detbegin{pmatrix}
                                      r & a & a & cdots &a \
                                      a & r& a & cdots & a \
                                      vdots & vdots& vdots & ddots & vdots \
                                      a & a & a & cdots & r
                                      end{pmatrix}=(r+(n-1)a)(r-a)^{n-1}$$







                                      share|cite|improve this answer














                                      share|cite|improve this answer



                                      share|cite|improve this answer








                                      edited Jan 30 at 19:36









                                      Martin Sleziak

                                      44.9k10122277




                                      44.9k10122277










                                      answered Jun 20 '13 at 18:08









                                      M.HM.H

                                      7,32211654




                                      7,32211654






























                                          draft saved

                                          draft discarded




















































                                          Thanks for contributing an answer to Mathematics Stack Exchange!


                                          • Please be sure to answer the question. Provide details and share your research!

                                          But avoid



                                          • Asking for help, clarification, or responding to other answers.

                                          • Making statements based on opinion; back them up with references or personal experience.


                                          Use MathJax to format equations. MathJax reference.


                                          To learn more, see our tips on writing great answers.




                                          draft saved


                                          draft discarded














                                          StackExchange.ready(
                                          function () {
                                          StackExchange.openid.initPostLogin('.new-post-login', 'https%3a%2f%2fmath.stackexchange.com%2fquestions%2f425597%2fprove-that-the-rank-of-1-i-is-n%23new-answer', 'question_page');
                                          }
                                          );

                                          Post as a guest















                                          Required, but never shown





















































                                          Required, but never shown














                                          Required, but never shown












                                          Required, but never shown







                                          Required, but never shown

































                                          Required, but never shown














                                          Required, but never shown












                                          Required, but never shown







                                          Required, but never shown







                                          Popular posts from this blog

                                          Can a sorcerer learn a 5th-level spell early by creating spell slots using the Font of Magic feature?

                                          Does disintegrating a polymorphed enemy still kill it after the 2018 errata?

                                          A Topological Invariant for $pi_3(U(n))$